Converting a first norm into a linear program












0












$begingroup$


I want to convert the following problem into a linear program.
$$ min_{x in mathbb{R}^n} qquad lvertlvert Ax - b rvertrvert_1$$



Here $A in mathbb{R}^{mtimes n}$, $x in mathbb{R}^n$ and $b in mathbb{R}^m$



This was my train of thought so far:



By definition, the first norm is,



$$ lvert lvert yrvert rvert_1 = lvert y_1rvert + lvert y_2 rvert + ...+ lvert y_nrvert$$
Now I know that an absolute value can be represented by replacing the variable with two non-negative variables. But I know it only for a single variable:
$$ lvert x rvert = x^+ - x^- $$
I have no idea how to do this when the expression is in the format $lvert a_i^Tx - b_irvert$ where $a_i^T$ is a row of the matrix $A$. (Yes I am dumb, I know it).



The best I could think of is adding a constraint $y = Ax -b$ and using $y$ as my variable:



begin{align}
min_{y_i^+, y_i^- in mathbb{R}} quad & y_1^+ + y_1^- + y_2^+ + y_2^- + y_3^+ + y_3^-+ dots + y_m^+ + y_m^-\
s.t. quad & a_1^Tx - b_1 = y_1^+ - y_1^- \
& a_2^Tx - b_2 = y_2^+ - y_2^- \
& a_3^Tx - b_3 = y_3^+ - y_3^- \
vdots
& a_m^Tx - b_m = y_m^+ - y_m^-
end{align}



This is all I can think. Is this the right way to go? Is there anything missing? Or is this entirely wrong? Any help would be appreciated.










share|cite|improve this question









$endgroup$

















    0












    $begingroup$


    I want to convert the following problem into a linear program.
    $$ min_{x in mathbb{R}^n} qquad lvertlvert Ax - b rvertrvert_1$$



    Here $A in mathbb{R}^{mtimes n}$, $x in mathbb{R}^n$ and $b in mathbb{R}^m$



    This was my train of thought so far:



    By definition, the first norm is,



    $$ lvert lvert yrvert rvert_1 = lvert y_1rvert + lvert y_2 rvert + ...+ lvert y_nrvert$$
    Now I know that an absolute value can be represented by replacing the variable with two non-negative variables. But I know it only for a single variable:
    $$ lvert x rvert = x^+ - x^- $$
    I have no idea how to do this when the expression is in the format $lvert a_i^Tx - b_irvert$ where $a_i^T$ is a row of the matrix $A$. (Yes I am dumb, I know it).



    The best I could think of is adding a constraint $y = Ax -b$ and using $y$ as my variable:



    begin{align}
    min_{y_i^+, y_i^- in mathbb{R}} quad & y_1^+ + y_1^- + y_2^+ + y_2^- + y_3^+ + y_3^-+ dots + y_m^+ + y_m^-\
    s.t. quad & a_1^Tx - b_1 = y_1^+ - y_1^- \
    & a_2^Tx - b_2 = y_2^+ - y_2^- \
    & a_3^Tx - b_3 = y_3^+ - y_3^- \
    vdots
    & a_m^Tx - b_m = y_m^+ - y_m^-
    end{align}



    This is all I can think. Is this the right way to go? Is there anything missing? Or is this entirely wrong? Any help would be appreciated.










    share|cite|improve this question









    $endgroup$















      0












      0








      0





      $begingroup$


      I want to convert the following problem into a linear program.
      $$ min_{x in mathbb{R}^n} qquad lvertlvert Ax - b rvertrvert_1$$



      Here $A in mathbb{R}^{mtimes n}$, $x in mathbb{R}^n$ and $b in mathbb{R}^m$



      This was my train of thought so far:



      By definition, the first norm is,



      $$ lvert lvert yrvert rvert_1 = lvert y_1rvert + lvert y_2 rvert + ...+ lvert y_nrvert$$
      Now I know that an absolute value can be represented by replacing the variable with two non-negative variables. But I know it only for a single variable:
      $$ lvert x rvert = x^+ - x^- $$
      I have no idea how to do this when the expression is in the format $lvert a_i^Tx - b_irvert$ where $a_i^T$ is a row of the matrix $A$. (Yes I am dumb, I know it).



      The best I could think of is adding a constraint $y = Ax -b$ and using $y$ as my variable:



      begin{align}
      min_{y_i^+, y_i^- in mathbb{R}} quad & y_1^+ + y_1^- + y_2^+ + y_2^- + y_3^+ + y_3^-+ dots + y_m^+ + y_m^-\
      s.t. quad & a_1^Tx - b_1 = y_1^+ - y_1^- \
      & a_2^Tx - b_2 = y_2^+ - y_2^- \
      & a_3^Tx - b_3 = y_3^+ - y_3^- \
      vdots
      & a_m^Tx - b_m = y_m^+ - y_m^-
      end{align}



      This is all I can think. Is this the right way to go? Is there anything missing? Or is this entirely wrong? Any help would be appreciated.










      share|cite|improve this question









      $endgroup$




      I want to convert the following problem into a linear program.
      $$ min_{x in mathbb{R}^n} qquad lvertlvert Ax - b rvertrvert_1$$



      Here $A in mathbb{R}^{mtimes n}$, $x in mathbb{R}^n$ and $b in mathbb{R}^m$



      This was my train of thought so far:



      By definition, the first norm is,



      $$ lvert lvert yrvert rvert_1 = lvert y_1rvert + lvert y_2 rvert + ...+ lvert y_nrvert$$
      Now I know that an absolute value can be represented by replacing the variable with two non-negative variables. But I know it only for a single variable:
      $$ lvert x rvert = x^+ - x^- $$
      I have no idea how to do this when the expression is in the format $lvert a_i^Tx - b_irvert$ where $a_i^T$ is a row of the matrix $A$. (Yes I am dumb, I know it).



      The best I could think of is adding a constraint $y = Ax -b$ and using $y$ as my variable:



      begin{align}
      min_{y_i^+, y_i^- in mathbb{R}} quad & y_1^+ + y_1^- + y_2^+ + y_2^- + y_3^+ + y_3^-+ dots + y_m^+ + y_m^-\
      s.t. quad & a_1^Tx - b_1 = y_1^+ - y_1^- \
      & a_2^Tx - b_2 = y_2^+ - y_2^- \
      & a_3^Tx - b_3 = y_3^+ - y_3^- \
      vdots
      & a_m^Tx - b_m = y_m^+ - y_m^-
      end{align}



      This is all I can think. Is this the right way to go? Is there anything missing? Or is this entirely wrong? Any help would be appreciated.







      norm linear-programming






      share|cite|improve this question













      share|cite|improve this question











      share|cite|improve this question




      share|cite|improve this question










      asked Dec 3 '18 at 20:16









      PPGoodManPPGoodMan

      657




      657






















          1 Answer
          1






          active

          oldest

          votes


















          1












          $begingroup$

          Seems fine. Don't forget to include the nonnegative constraints for your $y_i^+$ and $y_i^-$.



          Alternatively



          $$min sum_{i=m}^n z_i$$



          $$z_i ge a_i^Tx-b_i$$



          $$z_i ge -(a_i^Tx-b_i)$$



          $forall i in {1, ldots, m}$.






          share|cite|improve this answer











          $endgroup$













          • $begingroup$
            Thank you very much!
            $endgroup$
            – PPGoodMan
            Dec 4 '18 at 14:28











          Your Answer





          StackExchange.ifUsing("editor", function () {
          return StackExchange.using("mathjaxEditing", function () {
          StackExchange.MarkdownEditor.creationCallbacks.add(function (editor, postfix) {
          StackExchange.mathjaxEditing.prepareWmdForMathJax(editor, postfix, [["$", "$"], ["\\(","\\)"]]);
          });
          });
          }, "mathjax-editing");

          StackExchange.ready(function() {
          var channelOptions = {
          tags: "".split(" "),
          id: "69"
          };
          initTagRenderer("".split(" "), "".split(" "), channelOptions);

          StackExchange.using("externalEditor", function() {
          // Have to fire editor after snippets, if snippets enabled
          if (StackExchange.settings.snippets.snippetsEnabled) {
          StackExchange.using("snippets", function() {
          createEditor();
          });
          }
          else {
          createEditor();
          }
          });

          function createEditor() {
          StackExchange.prepareEditor({
          heartbeatType: 'answer',
          autoActivateHeartbeat: false,
          convertImagesToLinks: true,
          noModals: true,
          showLowRepImageUploadWarning: true,
          reputationToPostImages: 10,
          bindNavPrevention: true,
          postfix: "",
          imageUploader: {
          brandingHtml: "Powered by u003ca class="icon-imgur-white" href="https://imgur.com/"u003eu003c/au003e",
          contentPolicyHtml: "User contributions licensed under u003ca href="https://creativecommons.org/licenses/by-sa/3.0/"u003ecc by-sa 3.0 with attribution requiredu003c/au003e u003ca href="https://stackoverflow.com/legal/content-policy"u003e(content policy)u003c/au003e",
          allowUrls: true
          },
          noCode: true, onDemand: true,
          discardSelector: ".discard-answer"
          ,immediatelyShowMarkdownHelp:true
          });


          }
          });














          draft saved

          draft discarded


















          StackExchange.ready(
          function () {
          StackExchange.openid.initPostLogin('.new-post-login', 'https%3a%2f%2fmath.stackexchange.com%2fquestions%2f3024600%2fconverting-a-first-norm-into-a-linear-program%23new-answer', 'question_page');
          }
          );

          Post as a guest















          Required, but never shown

























          1 Answer
          1






          active

          oldest

          votes








          1 Answer
          1






          active

          oldest

          votes









          active

          oldest

          votes






          active

          oldest

          votes









          1












          $begingroup$

          Seems fine. Don't forget to include the nonnegative constraints for your $y_i^+$ and $y_i^-$.



          Alternatively



          $$min sum_{i=m}^n z_i$$



          $$z_i ge a_i^Tx-b_i$$



          $$z_i ge -(a_i^Tx-b_i)$$



          $forall i in {1, ldots, m}$.






          share|cite|improve this answer











          $endgroup$













          • $begingroup$
            Thank you very much!
            $endgroup$
            – PPGoodMan
            Dec 4 '18 at 14:28
















          1












          $begingroup$

          Seems fine. Don't forget to include the nonnegative constraints for your $y_i^+$ and $y_i^-$.



          Alternatively



          $$min sum_{i=m}^n z_i$$



          $$z_i ge a_i^Tx-b_i$$



          $$z_i ge -(a_i^Tx-b_i)$$



          $forall i in {1, ldots, m}$.






          share|cite|improve this answer











          $endgroup$













          • $begingroup$
            Thank you very much!
            $endgroup$
            – PPGoodMan
            Dec 4 '18 at 14:28














          1












          1








          1





          $begingroup$

          Seems fine. Don't forget to include the nonnegative constraints for your $y_i^+$ and $y_i^-$.



          Alternatively



          $$min sum_{i=m}^n z_i$$



          $$z_i ge a_i^Tx-b_i$$



          $$z_i ge -(a_i^Tx-b_i)$$



          $forall i in {1, ldots, m}$.






          share|cite|improve this answer











          $endgroup$



          Seems fine. Don't forget to include the nonnegative constraints for your $y_i^+$ and $y_i^-$.



          Alternatively



          $$min sum_{i=m}^n z_i$$



          $$z_i ge a_i^Tx-b_i$$



          $$z_i ge -(a_i^Tx-b_i)$$



          $forall i in {1, ldots, m}$.







          share|cite|improve this answer














          share|cite|improve this answer



          share|cite|improve this answer








          edited Dec 4 '18 at 8:13

























          answered Dec 4 '18 at 8:07









          Siong Thye GohSiong Thye Goh

          101k1466117




          101k1466117












          • $begingroup$
            Thank you very much!
            $endgroup$
            – PPGoodMan
            Dec 4 '18 at 14:28


















          • $begingroup$
            Thank you very much!
            $endgroup$
            – PPGoodMan
            Dec 4 '18 at 14:28
















          $begingroup$
          Thank you very much!
          $endgroup$
          – PPGoodMan
          Dec 4 '18 at 14:28




          $begingroup$
          Thank you very much!
          $endgroup$
          – PPGoodMan
          Dec 4 '18 at 14:28


















          draft saved

          draft discarded




















































          Thanks for contributing an answer to Mathematics Stack Exchange!


          • Please be sure to answer the question. Provide details and share your research!

          But avoid



          • Asking for help, clarification, or responding to other answers.

          • Making statements based on opinion; back them up with references or personal experience.


          Use MathJax to format equations. MathJax reference.


          To learn more, see our tips on writing great answers.




          draft saved


          draft discarded














          StackExchange.ready(
          function () {
          StackExchange.openid.initPostLogin('.new-post-login', 'https%3a%2f%2fmath.stackexchange.com%2fquestions%2f3024600%2fconverting-a-first-norm-into-a-linear-program%23new-answer', 'question_page');
          }
          );

          Post as a guest















          Required, but never shown





















































          Required, but never shown














          Required, but never shown












          Required, but never shown







          Required, but never shown

































          Required, but never shown














          Required, but never shown












          Required, but never shown







          Required, but never shown







          Popular posts from this blog

          Plaza Victoria

          In PowerPoint, is there a keyboard shortcut for bulleted / numbered list?

          How to put 3 figures in Latex with 2 figures side by side and 1 below these side by side images but in...